2
$\begingroup$

Cross-posting notice: This question is also posted on Math Stack Exchange: "Bounding a sum over primitive character"

I'm trying to obtain the upper bound of the sum $$ S= \sum_{\chi_s^{\star}} {\overline{\chi_s^{\star}}(\alpha)} \sum_{k=1}^{s} k\, \chi_s^{\star}(k) \prod_{p \mid t} \left( 1 - \chi_s^{\star}(p) \right), $$ where:

  • $s$ and $t$ are two coprime square-free integers,
  • $\alpha$ is an integer coprime to $s$, and
  • $\chi_s^{\star}$ denotes a primitive Dirichlet character modulo $s$.

Numerically, this sum appears to be much smaller than expected.
Question: Is it possible to obtain a sharp bound for this sum?

Below is my attempt:

  1. Attempt 1:

    By the Cauchy–Schwarz inequality, $$ \lvert S \rvert \le \left(\sum_{\chi_s^{\star}} \Bigl|\sum_{k=1}^{s} k\,\chi_s^{\star}(k)\Bigr|^2\right)^{\frac{1}{2}} \left(\sum_{\chi_s^{\star}} \Bigl|\sum_{d \mid t} \mu(d)\,\chi_s^{\star}(d)\Bigr|^2\right)^{\frac{1}{2}}, $$ which leads to, after some treating of the bilinear sum, the bound $$ \lvert S \rvert \le \frac{1}{\sqrt{3}}\, d(t)^{\frac{1}{2}}\, s^{\frac{3}{2}}\, \varphi(s) + o\Bigl(d(t)^{\frac{1}{2}}\, s^{\frac{3}{2}}\, \varphi(s)\Bigr). $$

    However, this bound is not sharp enough.

  2. Attempt 2:

    By the orthogonality of Dirichlet characters and Möbius inversion, $$ \begin{aligned} S &= \sum_{d \mid t} \mu(d) \sum_{k=1}^s k \sum_{\chi_s^{\star}} {\overline{\chi_s^{\star}}(\alpha)}\, \chi_s^{\star}(k d) \\ &= \sum_{d \mid t} \mu(d) \sum_{k=1}^s k \sum_{\substack{q \mid s \\ \alpha \equiv k d \, (\bmod\, q)}} \mu(q)\, \varphi(q), \end{aligned} $$ which leads to $$ S = \sum_{d \mid t} \mu(d) \sum_{k=1}^{s} k \prod_{p \mid (s,\, \alpha - k d)} \left(2 - p\right) $$ or $$ S = \sum_{q \mid s} \mu(q) \varphi(q) \sum_{d \mid t} \mu(d) \sum_{\substack{k=1 \\ (k,s)=1 \\ k \equiv d^{-1}\alpha \, (\bmod\, q)}}^{s} k. $$

    However, I have not been able to further treat either of these forms.

$\endgroup$
0

1 Answer 1

3
$\begingroup$

I think in attempt 2 the $\mu(q)$ should be $\mu(s/q)$, so after "which leads to" the $2-p$ should be $p-2$. For example if $\alpha=1$ the $d=1, k=1$ term should certainly be $\prod_{p\mid s} (p-2)$. In the corrected version:

$$ S = \sum_{d \mid t} \mu(d) \sum_{k=1}^{s} k \prod_{p \mid (s,\, \alpha - k d)} \left(p-2\right) $$

the sum over $k$ has no cancellation. So the only possible cancellation in the sum is the sum over $d$, which might sometimes have very little cancellation (e.g. if $t$ is prime.)

We can bound the inner sum $ \sum_{k=1}^{s} k \prod_{p \mid (s,\, \alpha - k d)} \left(p-2\right)$ by $s \sum_{k=1}^{s} \prod_{p \mid (s,\, \alpha - k d)} \left(p-2\right)$, and this should be sharp to within a factor of $2$. In this sum, since $d$ is coprime to $s$, $\alpha-kd$ hits each residue class modulo $s$ exactly once so this is the same as $s \prod_{p \mid s} ( p-2+ p-1) = \prod_{p \mid s} ( p (2p-3)) \leq \sigma_0(s) s^2$.

Then bounding trivially the sum over $d$ gives a bound of $\sigma_0(d) \sigma_0(s) s^2$ which is a little sharper than your Cauchy-Schwarz bound (which grows like $s^{5/2}$).

Is that sharp enough for your purposes? If not, is it really true that the sum empirically always has more cancellation than that?

$\endgroup$
1
  • $\begingroup$ Thanks! That was my oversight; however, I think it still leads to μ(s)∏(2−p), not ∏(p−2), which might allow for additional cancellation. Actually, this sum arises when calculating the remainder term R=∑g(s)S in a sieve method, where g is multiplicative and P(z)=s⋅t. The bound you proposed is indeed sharp, but it still doesn't let me control that remainder term directly; I'd need to account for the extra cancellation to bound it properly. $\endgroup$ Commented Feb 15 at 8:46

You must log in to answer this question.

Start asking to get answers

Find the answer to your question by asking.

Ask question

Explore related questions

See similar questions with these tags.